Difference between revisions of "1950 AHSME Problems/Problem 37"

(Created page with "==Probelm== If <math> y \equal{} \log_{a}{x}</math>, <math> a > 1</math>, which of the following statements is incorrect? <math>\textbf{(A)}\ \text{If }x=1,y=0 \qquad\\ \textbf...")
 
Line 8: Line 8:
 
\textbf{(D)}\ \text{If }0<x<z,y\text{ is always less than 0 and decreases without limit as }x\text{ approaches zero} \qquad\\
 
\textbf{(D)}\ \text{If }0<x<z,y\text{ is always less than 0 and decreases without limit as }x\text{ approaches zero} \qquad\\
 
\textbf{(E)}\  \text{Only some of the above statements are correct}</math>
 
\textbf{(E)}\  \text{Only some of the above statements are correct}</math>
 +
 +
==Solution==
 +
{{solution}}
 +
 +
==See Also==
 +
{{AHSME 50p box|year=1950|num-b=36|num-a=38}}
 +
 +
[[Category:Introductory Algebra Problems]]

Revision as of 08:38, 29 April 2012

Probelm

If $y \equal{} \log_{a}{x}$ (Error compiling LaTeX. Unknown error_msg), $a > 1$, which of the following statements is incorrect?

$\textbf{(A)}\ \text{If }x=1,y=0 \qquad\\ \textbf{(B)}\ \text{If }x=a,y=1 \qquad\\ \textbf{(C)}\ \text{If }x=-1,y\text{ is imaginary (complex)} \qquad\\ \textbf{(D)}\ \text{If }0<x<z,y\text{ is always less than 0 and decreases without limit as }x\text{ approaches zero} \qquad\\ \textbf{(E)}\  \text{Only some of the above statements are correct}$

Solution

This problem needs a solution. If you have a solution for it, please help us out by adding it.

See Also

1950 AHSC (ProblemsAnswer KeyResources)
Preceded by
Problem 36
Followed by
Problem 38
1 2 3 4 5 6 7 8 9 10 11 12 13 14 15 16 17 18 19 20 21 22 23 24 25 26 27 28 29 30 31 32 33 34 35 36 37 38 39 40 41 42 43 44 45 46 47 48 49 50
All AHSME Problems and Solutions